100 posts in the last 30 days

Is there a way to create a problem set based on a specific type of RC question? Based on my trends I need to work on meaning of phrase in context and application of hypothetical situations. Is it possible to create questions sets with that filter or are RC problem sets simply based on full passages.

User Avatar

Friday, Mar 03 2023

Delete.

Would the contrapositive of "If all farmers were to practice organic farming, they would be unable to produce enough food for Earth's growing population" be different from the contrapositive of "If farmers were to practice organic farming, they would be unable to produce enough food for Earth's growing population?"

The only difference is the "all" right before "farmers."

Thanks!

Hey 7Sage,

12 days until the Sept exam, WOOT! I am seeking advice on what PTs to do in the next two weeks. I have already done nearly

all the exams. My only completely untouched exams are old exams -- 39 and 44. Maybe also 10 and 13. Thus, I don't think I should focus on those in the next two weeks, although they are new to me. Correct me if I'm wrong.

I think I want to look at the newest PTs again, since they're nuanced compared to older tests. I have done 70-72 and 75-80 once each. I figure I will do 3 more before the 16th. Should I do 78-80? Does it matter? Any advice would be greatly appreciated. Thanks in advance.

hi, i am taking my lsat in june, and recently my game scores have plummeted for some reason, so i am trying to get my scores back up –– when i was doing ok on them, it was right after i did the games part of 7sage, and i guess i was doing like 2 at a time over and over again, but i am wondering if anyone who has super improved on the games noticed which strategy is better (doing the same game a thousand times in a row or like alternating 3 sections over and over again)?

Even though conditional logic seemed easy when I went through the CC, it continues to trip me up on harder questions, so I'm reviewing some basics:

To have a valid conditional statement, the SOME or MOST statement must come first followed by the ALL statement. (You can't conclude ANYTHING if the ALL statement comes before a SOME or MOST statement.) So A some B->C, therefore A some C is valid. Also A most B->C, therefore A most C is valid.

In a valid argument, the conclusion must be true. Basic valid argument examples include affirming the sufficient (A->B, A, therefore B ) ; denying the necessary (A->B, /B, therefore /A); transitive property (A->B->C, therefore A->C).

Other valid arguments show us instances where one thing (A) arrows out to two other things and there's therefore overlap between those two other things:

If A->B and A->C, then B some C.

If A->B and A some C, then B some C.

If A->B and A most C, then B some C.

If A most B and A most C, then B some C.

Invalid arguments mean the conclusion doesn't have to be true. Examples include affirming the necessary (A->B, B, therefore A); denying the sufficient (A->B, /A, therefore /B), and putting the ALL statement before the MOST or SOME statement (A->B some C, therefore A some C OR A->B most C, therefore A most C).

Other invalid arguments show us that we can't conclude anything from 2 some SOME statements:

A some B some C, therefore A some C (transitive property does NOT apply to SOME statements)

A most B most C, therefore A most C (transitive property does NOT apply to MOST statements)

A some B, A some C, therefore B some C.

You cannot take a contrapositive of a SOME or MOST statement.

Quantifiers have specific meanings on the LSAT that are often counterintuitive. Some is the trickiest one because it means at least one, but could be limited to one, and could go up to all. Be suspicious 😒 of ACs that use any variation of some, such as sometimes and somewhat. Always ask, “just one??” Many sucks too because is the same as some; it is not MOST. Few means some are, most are not. In most cases, we're talking about 3 or 4, but we can think of the range as up to 50 because more than that is MOST.

http://classic.7sage.com/lsat_explanations/lsat-61-section-4-question-14/

I simply had no clue for this one. And the correct AC seems to be making a huge assumption so I didn't choose it. Can we really reasonably infer that thieves who do not abandon cars before their owners notice that they have been stolen are more likely to get arrested and convicted? This just seem to be way too big of a gap, and frankly an unreasonable one, to be made.

Why do car thieves steal cars? To sell them or to keep them. So it's totally reasonable to assume that thieves DO NOT abandon the car right after stealing it. And I simply don't see how this makes them more likely to get caught as they could have driven really fast and got far away before the cops could do something.

Has anyone else had trouble with this question? What do you think?

Conservative: Socialists study history, and they do so to identify trends that inevitably lead to a socialist future. However, this undertaking is certain to fail, because it is only retroactively that historical trends appear inevitable.

Socialist: Socialists do indeed study history, but the purpose of this is practical rather than theoretical: Instead of trying to identify historical trends that themselves bring about socialism, socialists try to identify trends that inform the kind of work that socialists need to do to bring socialism about. Socialism thus is not the inevitable outcome of historical trends, it instead must be worked towards and deliberately brought about.

Under timed conditions this Point at Issue / Disagreement question had me genuinely confused: The conservative and the socialist agree in maintaining that socialists study historical trends, but they disagree about the purpose that these studies are supposed to serve: According to the conservative, these studies are a purely theoretical undertaking, the socialist deems them practical. This thus would have been the issue to anticipate.

The pertinent answer choices are (A) (“[A] socialist society is the inevitable consequence of historical trends that can be identified by an analysis of history”) and (E) (“Socialists analyze history in order to support the view that socialism is inevitable”).

In the case of (E), we do get at a version of the anticipated answer; (E) gets at the conservative’s portraying socialist analyses of history as purely theoretical undertakings, which the socialist rejects.

(A) is more tricky. If (A) said “Socialists believe that a socialist society is the inevitable consequence of historical trends that can be studied,” this arguably would be a right answer choice: The conservative does ascribe this view to socialists, the socialist does not. However, (A) is a claim in itself, not only a belief that socialists may or may not endorse. In this context, the situation is more straightforward: We have no reason to think that the conservative deems the creation of socialist societies inevitable, and the socialist explicitly denies that they are inevitable. So as it stands, the speakers actually seem to agree that (A) is false. This thus can’t be the point at issue.

Takeaways: It is crucial to distinguish clearly between the two viewpoints here, as well as between facts and beliefs. Do not interpret (A) as a belief that the conservative ascribes to socialists; it is rather a claim that the speakers themselves are supposed to endorse or reject.

PT16.S3.12 – Retina Scanners

This argument deals with retina scanners, machines that scan the blood vessel patterns in people’s eyes and stores these patterns, such that the scanners can recognized previously scanned patterns. The author furthermore posits that no two eyes have identical blood pattern vessels in their retinas, which seems to suggest that any given person has at least two such patterns, one for the left eye and one for the right one. The author then infers the conclusion that “[a] retina scanner can therefore be used successfully to determine for any person whether it has ever scanned a retina of that person before.”

We are supposed to identify a necessary assumption for this argument, i.e. an assumption that must be true for the conclusion to follow from the premises. Under timed conditions, I chose (B), which posits that everyone’s left and right eyes have identical patterns. I took this to be necessary for the conclusion to follow, due to conclusion’s scope (the conclusion is about “for any PERSON who ever had a retina scanned,” not about “for any given RETINA that ever has been scanned”). However, (B) seems to be false, for at least two reasons: (1) (B) goes against the information we get in the stimulus, where we are explicitly told that no two retinas have identical patterns. (2) (B) does not seem necessary for the rest of the claim that the conclusion seeks to establish (“Retina scanners allow you to answer the question, has one of the this person’s retinas ever been scanned?”). To make (B) a necessary condition, the conclusion would have to say something like “Even if you only scanned one of this person’s two retinas beforehand but not the other, retina scanners allow you to determine whether either of this person’s retinas has ever been scanned before.” However, (B) is not necessary for the way the conclusion is actually stated; the conclusion never says that the evidence to consider for any given person is a scan of only one of their retinas, as opposed to two.

The right answer choice (A) avoids this mistake by blocking another possible objection: What if people’s retina patterns change over time? Wouldn’t this make it impossible to recognize past scans later on, contrary to the argument’s conclusion suggests? (A) blocks this possible objection by establishing: Even if people get e.g. eye sicknesses, the patterns in their retinas remain unchanged over time.

Correct: A

Incorrect: C

https://classic.7sage.com/lsat_explanations/lsat-53-section-1-question-09/

"C" is incorrect because the male population could've stayed the same and the decline of the female population made it equal to the male population. "A" is correct because "proportional" gives relation to the whole population. It is saying that the decline of the female population is a decline in the total population. This takes it from being 2/3 of just females to 2/3 of the species.

Hi everyone,

I'm currently reviewing PT 37 that I completed yesterday and I'm on section 2 #12. Even watching JY's video, I'm still a little confused about the "cannot" and "without" in the stimulus. Anyway, I was just wondering-- what do you do when you have two indicators but they're in different groups? Or the way that JY presents it, maybe we're disregarding the "without" for being in group 3? And we're just treating it as a "no"?

I mean, either way, for the two indicators in one sentence issue--I feel like there was a lesson on this or that it was mentioned somewhere, but I can't seem to find it. If anyone could let met know or let me know what lesson it was in, that'd be great. And if you can further explain #12 I'd really appreciate it too! Thank you! :) Happy studying!

I can't understand why C is the right answer.

I mean if Stilgoe's case would have been stronger if he used these sources then would that it indicate that their view was more negative towards the railroad then the other writers?

Based on this reason, that's why I chose E, but it turns out E is wrong.

I'm stuck in this question.

Can anyone tell me how C is right and E can't be an answer?

Thanks!

Just want to share my thoughts and notes:

This formula right here: independence -> progress doesn’t warrant that more independence = more progress, so E is incorrect.

Cultures -> needs independence to replace dependence (natives replace outside imposition) -> progress.

A. anticipated answer choice

B. Staff and students are digging too deep, we’re only looking at cultures as a whole

C. Tailor is too details, not needed

D. Must is g2 so Advance -> prevent outsiders, not really align with the lawgic above.

Admin Note: Edited title. Please use the format: "PT#.S#.Q# - brief description of the question"

Hi Everyone!

I just completed the Logic Games lessons and now I'm ready to start drilling. I was wondering, how do you all approach drilling? Personally, I'm planning to do four games per day. Here's my routine: I tackle one drill, take a short break, and then watch the explanation. After that, I move on to the second drill, take another break, and watch the explanation. At the end of the day, I revisit both drills.

By the way, I have set the difficulty level of the drills to medium. Do you think that's the right level, or would you recommend a different difficulty?

User Avatar

Sunday, Aug 03

😖 Frustrated

Problems with Appeal Process

One of my favorite parts of this course was the opening statement about the mechanisms that ensure the test will have good questions, and we should therefore be less concerned about arbitrary distinctions. You say that there is a large board, review processes internally, and there are incentives for test takers to appeal problems since it will improve their scores.

I take issue with the last premise though. I took the LSAT a few months ago and had two really large issues with the administration.

For one thing it started 2-3 hours late and was administered terribly with lots of loud sounds and talking. I wrote a complaint to the LSAC about this and nothing happened. This hurts my confidence in the fairness of the test.

Second, I had a strong suspicion about one of the questions being incorrectly written, but since the questions aren't posted anywhere and there seemingly isn't a way to appeal for adjustments, I have a strong suspicion JY was wrong in his initial course. I do not think there is any way to appeal questions, which is such a shame since this is such an important test and it is developed by a private company with zero oversight and a profit incentive to crank questions out as cheaply as possible.

For this question, I chose E in the first round but switched to B in the blind review. I thought B would be a safer choice since the word "criticism"/"criticized" is used by both Murray and Jane. If Murray does not think it is wrong for politicians to accept gifts from lobbyists, why would they assume other politicians should have been criticized? Could someone please explain to me why B is not correct?

A lot of people who were part of the 170+ were able to reliably get -0 on the logic games section which gave them more room to make mistakes in the other sections. Someone could get -4 RC, -4 LR, and -0 LG and still get a 170. But now, with the section that people could reliably get -0 on removed, how will amount of questions needed to be correct to get to 170 change? Will the curve be more forgiving now that LG, the typical -0 section is gone, and the other sections that are less predictable and have more opportunities to make mistakes (especially RC) are still here?

Can someone explain how they were able to tease out the conditional elements in this stimulus?

I'm having trouble seeing how the second sentence fits into the conditional framework. It doesn't seem like it plays a role there.

All I can see with this, and it leads you straight to the answer, is:

Sentence 1: PPS-->MA

Conclusion: SA-->MA

Answer A: SA-->PPS

The negation didn't help me here since I didn't understand the conditional language at first, which is why I picked the wrong answer. If someone can explain why the negation would hurt the conclusion here, I'd be very grateful. Thank you.

https://classic.7sage.com/lsat_explanations/lsat-52-section-3-question-07/

Confirm action

Are you sure?